test #18 q.id = 667

77
Test #18 Q.Id = 667 Question 1 Of 46 Item Review Division : Dermatology A 40-year-old Caucasian woman comes to your office with concerns regarding a mole over her right leg. She has noticed a change in the color of the mole and she remembers the mole being much smaller. She noticed these changes 2 days ago while cleaning a bruise received while hiking. On examination, you notice a flat brown lesion with variegate pigmentation of black, blue and pink hues and it measures around 7.5mm with irregular borders. The histopathological examination of the lesion shows radial growth characterized by large number of atypical intraepithelial melanocytes arranged haphazardly at the dermal-epidermal junction. What is the diagnosis? A . Nodular melanoma B . Lentigo maligna melanoma C . Acral lentiginous melanoma D . Superficial spreading melanoma E . Amelanocytic melanoma Explanation: Superficial spreading melanoma is the most common sub type seen in approximately 70% of patients. It is most commonly located over the trunk in males and over legs in females. It presents as a flat or slightly elevated brown lesion with variegate pigmentation, measures more than 6mm with irregular asymmetric borders. The histopathological findings are characterized by increased number of intraepithelial melanocytes, which are atypical. Nodular melanoma is seen over the legs and trunk with rapid growth over weeks. It presents as dark brown to black papule or dome shaped nodule, which ulcerate and bleed with

Upload: drkhalid12

Post on 18-Nov-2014

604 views

Category:

Documents


6 download

TRANSCRIPT

Page 1: Test #18 Q.id = 667

Test #18   Q.Id = 667 Question 1 Of 46Item Review

Division : Dermatology

A 40-year-old Caucasian woman comes to your office with concerns regarding a mole over her right leg. She has noticed a change in the color of the mole and she remembers the mole being much smaller. She noticed these changes 2 days ago while cleaning a bruise received while hiking. On examination, you notice a flat brown lesion with variegate pigmentation of black, blue and pink hues and it measures around 7.5mm with irregular borders. The histopathological examination of the lesion shows radial growth characterized by large number of atypical intraepithelial melanocytes arranged haphazardly at the dermal-epidermal junction. What is the diagnosis?

  A. Nodular melanomaB. Lentigo maligna melanoma

  C. Acral lentiginous melanomaD. Superficial spreading melanoma

  E. Amelanocytic melanomaExplanation:

Superficial spreading melanoma is the most common sub type seen in approximately 70% of patients. It is most commonly located over the trunk in males and over legs in females. It presents as a flat or slightly elevated brown lesion with variegate pigmentation, measures more than 6mm with irregular asymmetric borders. The histopathological findings are characterized by increased number of intraepithelial melanocytes, which are atypical.

Nodular melanoma is seen over the legs and trunk with rapid growth over weeks. It presents as dark brown to black papule or dome shaped nodule, which ulcerate and bleed with minor trauma.

Lentigo maligna melanoma is seen in 4-15% of patients. It is located over the head, neck, and arms of fair skinned older individuals.

Acral lentiginous melanoma is the least common sub type and it occurs in palms, soles, or beneath nail plate in dark skinned individuals. The histopathology is similar to superficial spreading.

Amelanocytic melanoma is a non-pigmented melanoma appearing as pink or flesh colored and often mimics basal cell carcinoma or squamous cell carcinoma.

Educational Objective:Superficial spreading melanoma, the most common sub-type of malignant melanoma, presents as a flat or slightly elevated brown lesion with variegate pigmentation and the histopathology shows increased number of intraepithelial

Page 2: Test #18 Q.id = 667

atypical melanocytes.

33% of people answered this question correctly

Test #18   Q.Id = 658 Question 2 Of 46Item Review

Division : Dermatology

A 5-month-old infant is brought by his mother with complaints of rash for the past 15 days on his face, hands, and chest. She has tried a variety of over-the-counter products and many home remedies, as advised by her mother, but found no improvement. She also complains that the baby is constantly itching over these areas and she is having a hard time trying to keep his hands off. On physical examination, you see erythematous lesions on his cheeks with erosion, scaling, excoriated papules and plaques. Similar type of lesions is also found on his trunk, forehead and skull. The lesions are symmetrical with sparing of the diaper area. What is the most likely diagnosis?

  A. Contact dermatitis  B. Scabies  C. Exfoliative dermatitis

D. Atopic dermatitis  E. Seborrheic dermatitis

Explanation:

Atopic dermatitis typically manifests in infants of less than 6 months. The most common symptom is pruritus. These skin lesions usually start as erosions with serous exudates, intensely itchy papules, or plaques over erythematic skin as in this patient. In infants the lesions are usually symmetrical over cheeks, forehead, scalp, trunk, and extensor surfaces. Etiology is unknown, but anything that can dry the skin can exacerbate the condition. Prevention is the main stay of the treatment: avoid triggers; take short baths in lukewarm water; use moisturizer to skin. Acute attacks respond to low-moderate potency corticosteroid.

Contact dermatitis: History is not suggestive of this condition. Most common in adulthood, though irritative dermatitis is common in infants, most common is diaper dermatitis.

Scabies: Infestation arrives with close personal contact. Nocturnal pruritus is highly characteristic.

Exfoliative dermatitis: Usually these patients have prior dermatological condition, treatment with new medication, or underlying systemic illness. Individuals of more than 40yrs old are usually affected.

Seborrheic dermatitis: In infants it occurs with varied appearance from mild

Page 3: Test #18 Q.id = 667

patchy, scaling to wide spread thick, adherent crusts.

Educational Objective:Atopic dermatitis presents with pruritus and skin lesions which usually start as erosions with serous exudates, intensely itchy papules, or plaques over erythematic skin. In infants the lesions are usually symmetrical over cheeks, forehead, scalp, trunk, and extensor surfaces.

30% of people answered this question correctly

Test #18   Q.Id = 2012 Question 3 Of 46Item Review

Division : Dermatology

A 14-year-old white female presents to your office complaining of ‘spots’ on her face that make her feel embarrassed. Physical examination reveals comedones with minimal inflammation. Her past medical history is insignificant. She does not smoke or consume alcohol. What is the next best step in the management of this patient?

  A. Dietary modifications  B. Cream-based cleansers

C. Topical retinoids  D. Topical erythromycin  E. Oral doxycycline  F. Oral isotretinoin

Explanation:

It is important to assess the severity of the disease in patients presenting with acne. Comedones ("blackheads" and "whiteheads") with minimal inflammation represent mild disease. The best initial treatment in this patient is a topical retinoid started with low concentration. The effect of the treatment is usually slow, and the patient should be informed about that. Reactivation of the process or persistent mild acne may require adding a topical antibiotic (Choice D) or benzoyl peroxide to the regimen.

Oral antibiotics (Choice E) effective against Propionibacterium acnes are usually used if moderate-to-severe inflammation (papular and inflammatory acne) is present.

Oral isotretinoin (Choice F) is reserved for patients with nodulocystic and scarring acne.

Although there is some data indicative of the acne-promoting effect of refined carbohydrates, dietary modifications (Choice A) are not considered to be

Page 4: Test #18 Q.id = 667

effective in treating acne.

Cream-based cleansers (Choice B) should be avoided.

Educational Objective:In patients who present with non-inflammatory comedones (mild acne), topical retinoids are usually tried first. Oral antibiotics are usually used if moderate-to-severe inflammation (papular and inflammatory acne) is present. Oral isotretinoin is reserved for patients with nodulocystic and scarring acne.

13% of people answered this question correctly

Test #18   Q.Id = 677 Question 4 Of 46Item Review

Division : Dermatology

A 45-year-old woman comes to see you with the complaint of reddened areas over her face (flushed skin), which gets worse every time she drinks something hot or goes out in hot sunny weather. On examination, you notice erythema over her nose, cheeks, forehead and chin with telangiectasias, pustules and papules. Her vital signs are stable and she is afebrile. What is the most likely diagnosis?

  A. Acne vulgaris.  B. Seborrheic dermatitis.  C. Carcinoid syndrome.  D. Systemic lupus erythematosus.

E. Rosacea.Explanation:

The rosy hue with telangiectasia over the cheeks, nose, and chin is suggestive of rosacea. Typically hot drinks, heat, emotion, and other causes of rapid body temperature changes precipitate flushing. Episodes are usually intermittent but can progressively lead to permanently flushed skin. Sometimes papules and pustules may be present as in this patient. It most commonly occurs in patients between who are 30-60 years of age, fair skinned, with light hair and eye color. Pathogenesis is not known though hair follicle mites have been thought to play a role. Medical treatment only aims at the inflammatory papules, pustules, and erythema. Telangiectasias require laser surgery.

Choice (A): Acne vulgaris tends to occur in much younger individuals and comedones are usually present in acne vulgaris and are not seen in rosacea. Telangiectasia is not seen in acne.

Choice (B): Seborrheic dermatitis lacks pustules and papules; scales are usually present around the nose, eyebrows, ears and scalp.

Choice (C): Carcinoid syndrome patients experience flushing and may develop

Page 5: Test #18 Q.id = 667

telangiectasias too, but the flushing in rosacea tends to last much longer, whereas 20-30 sec flushing is characteristic of carcinoid. They also have diarrhea and cyanosis.

Choice (D): Systemic lupus erythematosus is associated with facial rash similar in distribution to rosacea but there are no papules and pustules; they will have other systemic complaints such as fever, arthralgias, mailase, renal involvement etc.

Educational Objective:Rosy hue with telangiectasia over the cheeks, nose, and chin is suggestive of rosacea that usually occurs in patients of 30-60 years of age.

40% of people answered this question correctly

Test #18   Q.Id = 681 Question 5 Of 46Item Review

Division : Dermatology  

A 45-year-old man comes with complaints of sudden onset of skin lesions, fever, and inability to eat or drink due to pain in his mouth and throat. The patient appears confused. His wife says that he was complaining of headache, malaise, and joint pain prior to developing the skin lesions. Generally, he has been in good health, other than an episode of urinary tract infection for which he was prescribed trimethoprim-sulfamethoxazole, about 10 days ago. On examination, the patient has altered level of consciousness and erythematous rash over the trunk. Cutaneous lesions are also present over the hands, arms and feet. The picture below shows some of these lesions. Oral mucosa shows erythema, blistering and ulceration. Inflammation of conjunctiva is noticed in both eyes. His vital signs are, PR: 92/min; BP: 110/80 mm of Hg; RR: 14/min; Temperature: 36.8C(98.4F). What is the most likely diagnosis?

A. Stevens -Johnson syndrome.  B. Erythema multiforme minor.  C. Staphylococcal scalded skin syndrome.  D. Toxic shock syndrome.  E. Impetigo.

Explanation:

Stevens Johnson syndrome is an immune complex mediated hypersensitivity reaction also known as erythema multiforme major. Drugs like sulfonamides, NSAIDs, and phenytoin most often cause it. The patient typically has sudden onset of mucocutaneous lesions. Mucosal involvement at 2 sites oral and conjunctival usually, is very typical. The typical lesions have the appearance of

Page 6: Test #18 Q.id = 667

‘target’ and are considered pathognomonic. These target lesions are shown in the picture above. The following signs are noted in these patients: fever, tachycardia, hypotension, altered level of consciousness, conjunctivitis, seizures, and coma. The patient should be treated similar to burns patient. Care should be given to hemodynamic stability, fluid status, and pain control. Treatment is primarily supportive.

Choice (B): Erythema multiforme minor is characterized by sudden onset of erythematous rash and target lesions and it usually follows herpes simplex infection. Mucosal involvement is rare, systemic symptoms are not so severe as in Stevens Johnson syndrome. This and Stevens Johnson syndrome comes under a spectrum of diseases.

Choice (C): Staphylococcal scalded skin syndrome is a disease usually seen in children younger than 6 years of age. It is a syndrome of acute exfoliation caused by toxins produced by staphylococcus aureus.

Choice (D): Toxic shock syndrome is an inflammatory response characterized by fever, rash, hypotension, constitutional symptoms, and multi-organ involvement. The rash typically appears in the trunk and then spreads to involve the extremities. Target lesions are not seen. Involvement of three or more organ systems is required for its diagnosis. It is usually associated with retained vaginal tampons.

Choice (E): Impetigo occurs commonly in children caused by coagulase positive staphylococcus aureus. It is characterized by red macules and papular lesions with honey colored crusts.

Educational Objective:Stevens Johnson syndrome is an immune complex mediated hypersensitivity and is characterized by sudden onset of mucocutaneous lesions as well as systemic signs of toxicity. Lesions are typically target shaped.

50% of people answered this question correctly

Test #18   Q.Id = 665 Question 6 Of 46Item Review

Division : Dermatology

An 80-year-old woman comes with complaints of a lesion on the neck. She requests removal of the lesion, saying that it’s greasy, unsightly and that she’s tired of people constantly staring at her neck. She sounds very upset. She says that the lesion has been there for quite a while and it has been gradually darkening with out much symptoms other than occasional itching. On examination, you see a lesion which is shown in the picture below. What is the diagnosis?

Page 7: Test #18 Q.id = 667

  A. Basal cell carcinoma  B. Malignant melanoma

C. Seborrheic keratosis  D. Actinic keratosis  E. Acrochordon

Explanation:

Seborrheic keratosis is also referred to as barnacles of old age. They become more common with advancing age and with no sex predilection, less common in population with dark skin. Seborrheic keratosis is usually asymptomatic but can be annoying; lesions can itch and rub occasionally. Lesions may vary in colors from pale brown to pink tones to dark brown or black and can occur anywhere in the body except palms and soles. Their natural history is slow enlargement with increasing thickness. Benign plaques of 3-20mm size, with velvety or warty, greasy surface and stuck on appearance are characteristic finding. Typical lesion is shown in the above picture. Shave biopsy provides material for histological diagnosis and removes lesion in a cosmetically acceptable manner.

Choice (A). Most common presentation is papule or nodule with a central scab or erosion. Occasionally the nodules have brown gray color or have stippled pigment

Choice (B). Changing mole is the most common symptom. Seborrheic keratosis may be difficult to distinguish from the lentigo maligna variant of malignant melanoma but the surface of the lesion in seborrheic keratosis is less lustrous and follicular orifices are plugged.

Choice (D). This is a pre-malignant condition, predisposing to squamous cell carcinoma. Lesions are dry, scaly and flat with an erythematous base.

Choice (E). Also called skin tags. They are flesh colored or hyperpigmented.

Educational Objective:Benign plaques of 3-20mm size, with greasy surface and stuck on appearance in an elderly person is characteristic for seborrheic keratosis. Extremely high yeild for USMLE.

25% of people answered this question correctly

Test #18   Q.Id = 683 Question 7 Of 46Item Review

Division : Dermatology

A 7-year-old girl presents with a rash all over her body. According to her mother, she was in good health until 4 days ago when she developed fever, cough and

Page 8: Test #18 Q.id = 667

eye pain. This morning she developed a rash on her face, which later spread all over her entire body. On examination, you notice erythematous maculopapular rash covering her entire body. Also, her buccal mucosa shows small red spots with bluish specks. Her vital signs are, PR: 86/min; RR: 14/min; BP: 110/70 mm of Hg; Temperature: 37.2C(99F). Which of the following is the diagnosis of her condition?

  A. Roseola infantum.  B. Rubella.  C. Varicella zoster infection.  D. Parvovirus infection.

E. Paramyxovirus infection.Explanation:

Measles is a highly contagious disease, caused by Paramyxovirus. Respiratory droplets transmit it. The following explains the order in which the symptoms and signs appear. Exposure to the virus; 10 days later prodrome of coryza, conjunctivitis, cough (remember 3 'C'); 2-3 days later appearance of Koplik’s spots; 24 hours’ later development of rash. Koplik’s spots appear as red spots with bluish specks located over the buccal mucosa, opposite the premolar tooth. The rash appears as erythematous maculopapular initially over the face and then it spreads to cover the entire trunk and extremities. Complications are pneumonia, vitamin A deficiency, and bronchiectasis. Late complications are immunosuppression and sub acute sclerosing panencephalitis.

Choice (A): Roseola infantum is an infection caused by herpes virus 6. The illness starts with fever, then after 4-5 days, a maculopapular rash appears over the trunk and spreads peripherally.

Choice (B): Rubella is characterized by erythematous, maculopapular rash with generalized lymphadenopathy and slight fever, polyarthritis and polyarthralgia.

Choice (C): Varicella zoster causes chicken pox. After a prodrome of fever, malaise, and anorexia a characteristic rash appears on the trunk and spreads peripherally. It rapidly develops into vesicles, which breaks to form scabs.

Choice (D): Parvovirus causes erythema infectiosum. It begins as erythema of cheeks, and then an erythematous rash appears on the arms and spreads to the trunk and legs.

Educational Objective:Measles, caused by paramyxovirus, is characterized by prodrome of cough, coryza and conjunctivitis, koplik’s spots, and maculopapular rash initially appearing on the face.

Page 9: Test #18 Q.id = 667

40% of people answered this question correctly

Test #18   Q.Id = 1969 Question 8 Of 46Item Review

Division : Dermatology

A 65-year-old man comes to your office with a six-month history of a non-healing ulcer on his right forearm. Physical examination demonstrates a scaling plaque with a central ulceration, 1.5 cm in diameter. Biopsy shows polygonal cells with atypical nuclei at all levels of epidermis with zones of keratinization. What is the single most important risk factor for this condition?

A. Sunlight  B. Arsenic  C. Aromatic hydrocarbons  D. Chronic osteomyelitis  E. Chronic scars

Explanation:

The clinical scenario described is typical for non-melanoma skin cancer. Histologic features in this patient suggest squamous cell carcinoma (SCC), the second most common form of non-melanoma skin cancer after basal cell carcinoma. Squamous cell carcinoma is more aggressive than basal cell carcinoma, because it metastasizes frequently. Exposure to sunlight is considered the single most important factor in the development of SCC. The ultraviolet spectrum of sunlight causes DNA damage and is mutagenic. People with only a little amount of pigment in their skin (fair complexion, blue eyes, tendency to sunburn easily) are particularly prone to SCC.

Arsenic (Choice B) and aromatic hydrocarbons (Choice C) are other well-known factors associated with SCC, but they have much less overall importance than sunlight.

Intrinsic factors predisposing to SCC include chronic osteomyelitis (Choice D) and chronic scars (Choice E), but they are uncommon causes of this condition.

Educational Objective:Squamous cell carcinoma is the second most common form of non-melanoma skin cancer. The single most important risk factor for the development of squamous cell carcinoma is exposure to sunlight.

43% of people answered this question correctly

Test #18   Q.Id = 1947 Question 9 Of 46Item Review

Division : Dermatology

A 45-year-old male comes to you with complaints of throbbing pain over the

Page 10: Test #18 Q.id = 667

pulp of his right index finger for the last two days. He says that he had been feeling warm also for the last three days. On examination, he has swollen, soft, and tender distal pulp space of the right index finger with some non-purulent vesicles. The picture of his hand is shown below. Tzanck smear of vesicles show multinucleated giant cells. He has never had any sexual transmitted disease or cold sores in the past. Which of the following is most likely the occupation of this patient?

A. A dentist  B. Commercial sex worker  C. A gardener  D. A tailor

Explanation:

The clinical presentation of this patient is suggestive of herpetic whitlow. Herpetic whitlow is the most common viral infection of the hand caused by either type 1 or 2 herpes simplex virus and is usually self-limiting. Direct inoculation of the virus through the broken skin is the mode of infection. It is most commonly seen in women with genital herpes or children with herpetic gingivostomatitis; however, health care workers are also at increased risk of this infection, due to contact with infected serum or saliva (Choice A). About 14% of adult cases of herpetic whitlow are health care workers.

Patients with herpetic whitlow often present with throbbing pain in the distal pulp space, which is swollen, soft and may be tender. Lateral nailfold may also be affected and non-purulent vesicles on volar aspects are clinically diagnostic. Diagnosis is confirmed by the presence of a history of exposure and multinucleated giant cells on Tzanck smear of vesicles. Systemic symptoms, like fever and lymphadenopathy, may occur. This is a self-limiting illness; however, oral acyclovir and topical bacitracin to prevent secondary infection may be used.

(Choice B) Commercial sex workers with genital herpes or oral herpes can develop whitlow; however, this patient has no history of either.

(Choice C) Gardeners are at increased risk of fungal infection, like sporotrichosis, or bacterial infection, like felon, due to skin injury with thorns; however, multinucleated giant cells on Tzanck smear of a lesion is suggestive of herpetic whitlow in this patient.

(Choice D) Tailors can develop felon due to needle injuries. It is important to distinguish felon from whitlow. Felon is a bacterial infection of the distal volar space, characterized by tense abscess and intense throbbing pain. Incision and

Page 11: Test #18 Q.id = 667

drainage with appropriate antibiotic (e.g., cephalosporins) is the treatment of choice.

Educational Objective: Herpetic whitlow is the most common viral infection of the hand, caused by either type 1 or 2 herpes simplex virus and is self-limiting. Health care workers who come in direct contact with orotracheal secretion are at increased risk of developing whitlow.

*Extremely high yield question for USMLE!!!

 Image is reproduced with permission from Dr.Samuel Freire da Silva, M.D.

17% of people answered this question correctly

Test #18   Q.Id = 650 Question 10 Of 46Item Review

Division : Dermatology

A 23-year-old college student comes to see you with complaints of itching all over her body for the past 10 days. She hardly gets to sleep at night, because of it. On further questioning, she agrees that her roommate is also suffering from similar complaints. On examination, you notice vesicles and pustules in short, gray wavy channels on the finger webs, heels of palms and in wrist creases. Papules are also seen over nipples and areola of breasts. Her vitals are stable and she is afebrile. What is your diagnosis?

  A. Insect bites  B. Urticaria

C. Scabies  D. Body lice  E. Bed bugs

Explanation:

Scabies is caused by infestation with Sarcoptes scabiei; it usually spares the head and neck area. It presents with generalized itching. Pruritic vesicles and pustules in “runs”, especially in finger webs, heels of palms and in wrist creases, are seen. Pruritic papules are seen over nipples and areola in females; over scrotum and penis in males. Examination of scrapes from excoriated lesions under light microscope reveals mites, ova, and feces. Bedding and clothing should be cleaned or set aside for 2wks. Lindane as 1% cream or lotion applied from neck down overnight may be used in adults. The dermatitis can be controlled with low potency topical steroids.

Choice (A): Insect bites typically appear as pruritic papules, grouped in the bite area. Vesicular and bullous bite reactions are common.

Page 12: Test #18 Q.id = 667

Choice (B): Urticaria occurs as evanescent wheals or hives with intense itching. Most incidents are acute, self-limited and can result from immunologic or non-immunologic basis.

Choice (D): Body lice infect the seams of clothing. Nits are found in the seams, not on human hairs. Maculae caerulea, hemosiderin stained purpuric spots where lice have fed, is diagnostic of body lice infestation.

Choice (E): Bed bugs can be seen anywhere but tends to occur often in old furniture and hide in seams and folds of mattresses. The bites are painless but pruritus and purpuric macules may appear. Bites are noted in groups of threes over exposed areas.

Educational Objective:Scabies is highly contagious disease and it presents with generalized itching and pruritic papules over penis and scrotum in males and areolas and breasts in females.

75% of people answered this question correctly

Test #18   Q.Id = 673 Question 11 Of 46Item Review

Division : Dermatology

A 64-year-old female, a known patient of yours, presents with complaints of lesions over her breasts and thighs. She says that she had been experiencing severe pain in those areas prior to developing redness and blisters. She is a known patient of valvular heart disease with atrial fibrillation. She started having treatment for atrial fibrillation about, 4 days ago, with antiarrhythmics and oral anticoagulants. Her other medical problems include ulcerative colitis, diagnosed 20 years ago and a resection of part of her colon. On examination, you notice well-demarcated lesion with bullae and necrotic changes over her thighs and breasts. Her vital signs are, PR: 82/min; BP: 140/90 mm of Hg; RR: 14/min; Temperature: 36.8C (98.4F). What is the most likely diagnosis?

  A. Necrotizing fasciitis.  B. Venous gangrene.

C. Warfarin induced necrosis.D. Pyoderrma gangrenosum.

  E. Cholesterol embolisation syndrome.Explanation:

Warfarin induced skin necrosis is a serious complication of oral anticoagulants. Protein C deficiency is sometimes associated with this condition. Females are most commonly affected; the common sites involved are breasts, buttocks, thighs, and abdomen. The initial complaint is pain followed by bullae formation and skin necrosis. It mostly occurs within weeks after starting therapy. One

Page 13: Test #18 Q.id = 667

should promptly administer vitamin K in the early stages of the lesion; if the lesion had progressed then warfarin should be discontinued. Heparin should be used to maintain anticoagulation until the necrotic lesions heal. Few patients require skin grafting.

Choice (A): Necrotizing fasciitis is a rapidly spreading infection involving fascia of deep muscles. It occurs after trauma or recent surgery. Typically, there is a history of sudden onset of pain and swelling, which progresses to purplish discoloration of the injured area with bullae and serosanguineous discharge.

Choice (B): Venous gangrene usually affects distal part of limb, appendix, small intestine, and rarely organs such as gall bladder, pancreas and testis. It is characterized by poor or absent peripheral pulse, venous return and capillary response to pressure. The involved area changes color from pale gray, finally to greenish black or black.

Choice (D): Pyoderma gangrenosum is an ulcerative skin lesion. The initial lesion is often described by the patient as a bite like reaction with a small papule or pustule. Pain is the main complaint; malaise and arthralgia may also be present.

Choice (E): Cholesterol embolization syndrome should be suspected in patients who develop worsening renal function, hypertension or distal ischemia, following invasive arterial procedure. Livedo reticularis can be seen on skin examination.

Educational Objective:Warfarin induced skin necrosis presents with pain followed by bullae formation and skin necrosis and it commonly involves breasts, buttocks, thighs, and abdomen.

46% of people answered this question correctly

Test #18   Q.Id = 661 Question 12 Of 46Item Review

Division : Dermatology

A 50-year-old man comes in for his annual check up. You notice a swelling on his left eyelid which he casually acknowledges by saying that it comes on and off for the past 2 years. It initially started as a painful lesion which regressed with hot compresses and reappeared. On examination of his left eye you see a nodular, painless, rubbery lesion on the eyelid, without any discharge, redness or other abnormalities. What is your next step in the management of his eye lesion?

  A. Anti staphylococcal antibiotics  B. Direct steroid injection

C. Incision and curettage

Page 14: Test #18 Q.id = 667

  D. Frequent hot compressesE. Histopathological examination

Explanation:

The initial presentation of a painful swelling progressing to a nodular rubbery lesion is typical of chalazion. It’s a granulomatous condition that develops when a meibomian tear gland of the eyelid becomes obstructed. Persistent or recurrent chalazion may be due to squamous cell carcinoma. So, it requires a histopathologic examination to rule out malignancy.

Choice (A) is not indicated, as it’s a granulomatous condition.

Choice (B) and (C) are for symptomatic patients not responding to other modes of treatment.

Choice (D) is for large lesions; hot compresses may allow them to drain.

Educational Objective:Recurrent chalazion requires histopathologic examination because of the risk of underlying squamous cell carcinoma.

18% of people answered this question correctly

Test #18   Q.Id = 646 Question 13 Of 46Item Review

Division : Dermatology

A 28-year-old Caucasian male presents with a rash on his trunk. He complains of constant itching over the area. On examination, only significant findings are 4 circular patches with central clearing and a scaly border, measuring around 3-8cms in diameter. He has no other medical problems. He denies any family history of diabetes. He admits having sex with 2 sexual partners without using condoms. His vital signs are PR: 84/min; BP: 120/80 mm of Hg; RR: 14/min; Temperature: 37C(98.4F). What is your diagnosis?

A. Tinea corporis infection  B. Psoriasis  C. Erythema multiforme  D. Pityriasis rosea  E. Secondary syphilis

Explanation:

Ring-shaped scaly patches with central clearing and distinct borders is suggestive of tinea corporis. Any species of dermatophyte may cause this condition but Trichophyton rubrum is the most frequent culprit. Itching is the most predominant symptom. Microscopic examination uaing KOH reveals hyphae, which confirms the diagnosis. Topical treatment with 2% antifungal lotions and creams (terbinafine) or systemic treatment with griseofulvin (for

Page 15: Test #18 Q.id = 667

extensive disease) offers good relief. Patients with extensive disease should be investigated for underlying immunosuppression, which include DM, HIV etc.

Choice (B): Psoriasis has the typical salmon colored patches with silvery scaling and peeling over extensor surfaces of elbows, knees and scalp.

Choice (C): Erythema multiforme does not have a peripheral scaly border and is mostly acral in distribution. It is often associated with a recent herpes simplex infection.

Choice (D): Pityriasis rosea lesions are numerous. They are oval and scaly plaques, and follow the cleavage lines of the trunk. The centers of the lesion have a crinkled, cigarette paper like appearance. It often presents with an initial lesion called the herald patch much larger than the later lesions.

Choice (E): Secondary syphilis skin lesions are usually maculopapular; involvement of the palms and soles is characteristic. Mucous membrane lesions may present as ulcers, papules or diffuse redness. History of multiple sexual partners is a distractor.

Educational Objective:Rash of tinea corporis is often pruritis and is in the form of ring-shaped scaly patches with central clearing.

39% of people answered this question correctly

Test #18   Q.Id = 674 Question 14 Of 46Item Review

Division : Dermatology  

A 25-year-old male presents with complaints of skin lesions, accompanied with severe itching and burning over his knees, elbows and back of his neck. He also gives a history of loose stools, flatulence, weight loss of more than 10 lbs and fatigue for the past 6 months. On examination, you notice papules and vesicles over the extensor aspect of his elbows, knees, posterior neck and scalp. Lab work shows presence of anti-endomysial antibodies. His vital signs are stable. Which of the following is the most likely diagnosis?

  A. Erythema multiforme.  B. Bullous pemphigoid.

C. Dermatitis herpetiformis.  D. Kaposi’s sarcoma.  E. Acantholytic dermatosis.

Explanation:

The presence of pruritic papules and vesicles over extensor surfaces of elbows,

Page 16: Test #18 Q.id = 667

knees, posterior aspect of neck and scalp in a patient with gastrointestinal symptoms, suggestive of malabsorption are typical of dermatitis herpetiformis. The presence of anti-endomysial antibodies is also characteristic of this condition. These patients also suffer from celiac sprue or gluten sensitive enteropathy. There is an increased risk of gastro-intestinal lymphomas, which can be reduced by following a gluten free diet.

Choice (A): Erythema multiforme is characterized by sudden onset of symmetric erythematous skin lesions. Target skin lesions with clear center and concentric erythematous rings may be noted. Herpes simplex infection, systemic diseases, and drug reactions are often associated with this condition.

Choice (B): Bullous pemphigoid is a pruritic disease, characterized by tense blisters in flexural areas. The affected patients are usually over the age of 60.

Choice (D): Kaposi’s sarcoma is a rare malignant skin lesion seen in patients with AIDS. It appears as red, purple or dark plaques or nodules on cutaneous and mucosal surfaces. It is managed aggressively with intralesional chemotherapy, radiation, and systemic chemotherapy.

Choice (E): Acantholytic dermatosis is also known as Grover disease. Patients present with pruritus and erythematous to brown keratotic papules over the anterior chest, upper back, and lower rib cage. Etiology is not known.

Educational Objective:Dermatitis herpetiformis is suggested by the presence of pruritic papules and vesicles over extensor surfaces and by the presence of anti-endomysial antibodies in the serum.

53% of people answered this question correctly

Test #18   Q.Id = 669 Question 15 Of 46Item Review

Division : Dermatology

A 65-year-old man presents with complaints of pain and rash with blisters, over the left side of his chest. He says that he had pain over the area for the past 2 days. This morning he noticed blisters while changing his shirt. He also complains of malaise and headache. On examination, you notice grouped tense vesicles as a band along the left side of his chest. His vitals are as follows: PR: 82/min; BP: 140/90 mm of Hg; RR: 14/min; Temperature: 36.8C(98.4F). Which of the following is the etiology of his condition?

  A. Herpes simplex virusB. Varicella zoster

  C. Poison ivy

Page 17: Test #18 Q.id = 667

  D. Human papilloma virus  E. Poxvirus

Explanation:

The above presentation is a classic presentation of shingles caused by varicella zoster virus. Usually pain precedes the vesicular eruption by 48 hours or more. The eruption occurs in a band-like fashion along the affected dermatome. Following VZV infection that causes chicken pox, latent infection is established in sensory dorsal root ganglia; reactivation of the latent infection called shingles is caused by stress, old age, fatigue, cancer, radiation therapy, and injury. The complications are persistent neuralgia, anesthesia, or scarring of affected areas. Nerve palsies and encephalitis may occur.

Choice (A): Herpes simplex virus infection usually presents as small grouped vesicles on an erythematous base located over the oro-labial and genital areas. Regional lymphadenopathy is present. Dermatomal distribution is quite atypical for Herpes simplex.

Choice (C): Poison-ivy can cause contact dermatitis in a streak, after single brushing with the plant and is pruritic, whereas shingles is painful.

Choice (D): Human papilloma virus causes warts, which appear as verrucous papules anywhere on the skin or mucous membranes. These lesions are asymptomatic.

Choice (E): Poxvirus causes molluscum contagiosum, which appears as multiple, rounded, dome shaped papules with central umbilication, mainly involving the face, lower abdomen and genitalia.

Educational Objective:Shingles caused by varicella zoster virus is characterized by vesicular eruption that occurs in dermatomal distribution and is often preceded by pain.

66% of people answered this question correctly

Test #18   Q.Id = 656 Question 16 Of 46Item Review

Division : Dermatology

A 9-year-old boy is brought in by his mother with complaints of itching, burning and oozing skin lesions on both of his legs. The boy appears tanned. When you ask if he has been spending time outdoors, he talks with great excitement about his camping trip in the woods with his dad, from which he just returned yesterday. On examination, you notice typical linear streaked vesicles on both his lower limbs with erythema, edema and weepy-crusted lesions. What type of reaction is responsible for these lesions?

Page 18: Test #18 Q.id = 667

  A. IgE mediated hypersensitivity  B. Antibody mediated hypersensitivity  C. Immune complex mediated hypersensitivity

D. Cell mediated hypersensitivityExplanation:

The history, and the clinical features, are all classic presentations of allergic contact dermatitis. The patient appears to have allergic reactions to poison oak or ivy. The typical linear streaked vesicles are suggestive of this. Allergic contact dermatitis presents with erythema, edema, pruritus, tiny vesicles and weepy and crusted lesions. This is an example of type IV hypersensitivity, also called delayed-type hypersensitivity or cell-mediated hypersensitivity. The allergen causes dermal inflammation on direct contact with the skin. The reaction occurs after 24-48 hours from the time of contact.

Choice (A). IgE mediated hypersensitivity is also called type I hypersensitivity. IgE antibodies attached to mast cells and within minutes after allergen exposure the mediators are released. Atopy and anaphylaxis are the two sub groups.

Choice (B). Antibody mediated hypersensitivity is also called type II hypersensitivity. Cytotoxic reactions involve specific reactions of IgG or IGM to cell bound antigens. Examples of these types of reactions are immune hemolytic anemia and Rh hemolytic disease in newborn.

Choice (C). Immune complex mediated hypersensitivity is also called type III hypersensitivity. Antibodies of IgG or IGM forms complexes with allergens and activates complement cascade. Examples of this reaction are serum sickness and Arthus reaction.

Educational Objective: Allergic contact dermatitis presents with erythema, edema, pruritus, tiny vesicles and weepy and crusted lesions 24-48 hours after the contact with the allergen and is due to cell-mediated hypersensitivity.

43% of people answered this question correctly

Test #18   Q.Id = 648 Question 17 Of 46Item Review

Division : Dermatology

A 28-year-old male comes to your office with complaints of multiple asymptomatic skin lesions for the past 1 month. He denies any systemic features. The picture below shows the lesions present over his abdomen. Similar lesions are also present over his inner thighs, pubic area and genitals. What is your diagnosis?

Page 19: Test #18 Q.id = 667

  A. Warts  B. Lichen planus

C. Molluscum contagiosum  D. Dermatitis herpetiformis  E. Miliaria

Explanation:

Molluscum contagiosum is caused by poxvirus. It presents as single or multiple rounded, dome shaped papules with central umbilication. The image shown above shows all these features. Initially the lesions are firm, later they may soften and suppurate. There are no systemic signs and lesions are asymptomatic. The sites involved in adults are the face, lower abdomen, and genitals. In children, trunk and extremities are mainly involved. In sexually active individuals the lesions are confined to the penis, pubis and inner thighs as in this patient. It is considered a sexually transmitted disease. It is also seen in AIDS patients, especially when the CD4 count is less than 100/ul. These patients usually have extensive involvement of the face, neck, and genital areas. It is very difficult to treat. The best form of treatment is curettage or application of liquid nitrogen. Choice (A): Warts are caused by HPV. The lesions appear as verrucous papules and occur anywhere on the skin or mucous membrane. Genital HPV has been associated with risk for cervical cancer.

Choice (B): Lichen Planus is an inflammatory condition characterized by pruritic, violaceous, flat-topped papules with fine white streaks on the surface called Wickham’ s striae.

Choice (D): Dermatitis herpetiformis is an uncommon disease characterized by pruritus, papules, and vesicles mainly over the elbows, knees, buttocks, posterior neck, and scalp. It is associated with HLA B8, -DR3, and DQw2 and seen in patients with gluten sensitive enteropathy.

Choice (E): Miliaria is also known as heat rash. It presents as superficial aggregated small vesicles, papules, or pustules over the trunk, which are associated with burning and itching. It is commonly seen in people living in hot and moist climates.

Educational Objective:Molluscum contagiosum presents as single or multiple rounded, dome shaped papules with central umbilication.

66% of people answered this question correctly

Page 20: Test #18 Q.id = 667

Test #18   Q.Id = 676 Question 18 Of 46Item Review

Division : Dermatology

A 55-year-old man presents with sudden onset of blisters all over his body. He complains of pain in the involved areas and had first noticed the lesions in his mouth a few days ago. On examination, you notice crops of flaccid bullae over normal appearing skin and large erosions at sites where the bullae had ruptured. The oral mucosa shows erosions and ulcerations. Slight rubbing of the uninvolved skin causes easy separation of the epidermis. Immunofluorescence microscopy shows deposits of IgG intracellularly in the epidermis. His vital signs are, PR: 82/min; BP: 140/90 mm of Hg; RR: 14/min; Temperature: 36.8C(98.4F). What is the most probable diagnosis?

  A. Bullous pemphigoid.  B. Bullous impetigo.

C. Pemphigus vulgaris.  D. Erythema multiforme.  E. Dermatitis herpetiformis.

Explanation:

Pemphigus is a blistering disease occurring in skin and mucous membranes. The etiology is unknown. It is characterized by flaccid bullae that appear spontaneously and are tender and painful when they rupture. Often lesions first appear in the oral mucous membrane, which rapidly becomes erosive. Easy separation of epidermis, on superficial pressure is Nikolsky’s sign and is positive in pemphigus. Immunofluorescence microscopy reveals IgG deposits intercellularly in the epidermis, which is characteristic of pemphigus. Commonly used drug in treatment is steroids. Immunosuppressive agents like azathioprine may be used with prednisone and methotrexate.

Choice (A): Bullous pemphigoid is a benign pruritic disease, characterized by tense blisters and not flaccid, as in pemphigus. Oral lesions are very rare. Immunofluorescence shows IgG and C3 deposits in the dermal epidermal junction.

Choice (B): Bullous impetigo is a contagious infection of the skin caused by staphylococcus or streptococcus or both. The bullous type is associated with staphylococcus aureus infection. The lesions consist of macules, vesicles, bullae, and honey colored crusts, which when removed leave red denuded areas.

Choice (D): Erythema multiforme lesions can be macular, papular, bullous or purpuric. “Target” lesions can be noted and they favor the extensor surface, palms, soles, and mucous membranes.

Choice (E): Dermatitis herpetiformis is manifested by pruritic papules, vesicles over the elbows, knees, buttocks, posterior neck, and scalp.

Page 21: Test #18 Q.id = 667

Immunofluorescence shows granular IgA deposits along dermal papillae. Circulating anti-endomysial antibodies can be detected in all patients. These patients can have gluten sensitive enteropathy.

Educational Objective:Pemphigus vulgaris is a mucocutaneous blistering disease and is characterized by flaccid bullae and intracellular IgG deposits in the epidermis.

41% of people answered this question correctly

Test #18   Q.Id = 651 Question 19 Of 46Item Review

Division : Dermatology

A 65-year-old male comes into your office on a hot summer afternoon. He complains of blisters and intense itching all over his body for the past 2 days. On further questioning, he mentions that he has been having itchy red swelling all over for the past 2 months, which he thought was due to the summer heat. On examination, lesions are seen on both normal and erythematous skin over flexural areas, groin, axilla and legs. One of such lesions is shown in the image below.  His PR: 82/min; BP: 140/80 mm of Hg; RR: 14/min; Temperature: 36.8C(98.4F). Which of the following will be seen in the above-described condition?

A. IgG and C3 deposits at the dermal-epidermal junction.  B. IgG deposits intracellularly in the epidermis.  C. IgG deposits in a linear band at the dermal-epidermal junction.  D. C3 at the basement membrane zone.  E. Intradermal edema with leukocyte infiltration.

Explanation:

The above image shows a tense blister on leg. The features of tense blisters in flexural areas with no oral lesions with an antecedent history of months of urticaria in an elderly patient of more than 60 year old are characteristic of bullous pemphigoid. It usually presents with pruritus and is precipitated by ultra-violet rays, NSAIDs, and antibiotics. The pathophysiology is IgG binding to skin basement membrane, which activates compliment and releases the inflammatory mediators. Immunofluorescence microscopy reveals diagnostic finding of deposits of IgG and C3 at the dermal-epidermal junction.

Choice (B): This is a characteristic feature of pemphigus vulgaris. It is an intra-epidermal blistering disease with autoantibodies to adhesion molecules expressed in skin and mucous membranes. The bullae are usually flaccid.

Page 22: Test #18 Q.id = 667

Choice (C): Is a feature of cicatricial pemphigoid, an auto immune blistering disease, that affects mucous membranes including mouth, oropharynx, conjunctiva, nares, and genitalia.

Choice (D): Is a feature of herpes gestationis. It usually begins in the second trimester of pregnancy and is characterized by sub epidermal blisters.

Choice (E): Is a feature of urticaria.

Educational Objective:IgG and C3 deposits at the dermal-epidermal junction are diagnostic of bullous pemphigoid. Know the pathology of pemhigoid and pemphigus vulgaris thoroughly. Very hot topic for USMLE.

26% of people answered this question correctly

Test #18   Q.Id = 682 Question 20 Of 46Item Review

Division : Dermatology

An 8-year-old boy is brought with the complaint of itchy rash with blisters over his face, trunk and legs for the past 2 days. On examination, you notice macules, pustules, vesicles, and honey colored crusts around mouth, nose, legs, buttocks and trunk area. His vital signs are stable but he is febrile (Temperature: 37.7C(100F)). What is the most likely diagnosis?

  A. Contact dermatitis.B. Impetigo.

  C. Herpes simplex infection.  D. Erythema multiforme.  E. Varicella zoster infection.   F. Erysipelas.

Explanation:

Impetigo is a contagious disease caused by staphylococcus aureus or streptococcus, or both. It presents as an erythematous macule, which then rapidly evolves into vesicles and pustules. The pustules later rupture leaving honey colored crusted exudates. There are 2 forms of impetigo, vesiculo-pustular and bullous types. Bullous type is caused by staphylococcus.

Choice (A): Contact dermatitis is an inflammatory response to antigen or irritants; the patients usually give history of exposure to the offending irritant. They present with pruritic, red, edematous skin with bullae, which later becomes exudative. A wheal and flare response may be seen at the site of exposure.

Choice (C): Herpes simplex infection presents as small grouped vesicles over erythematous skin especially in oro-labial and genital areas. It usually follows

Page 23: Test #18 Q.id = 667

minor infections, trauma, as well as stress or sun exposure.

Choice (D): Erythema multiforme is characterized by the sudden onset of erythematous rash with target lesions. It usually follows infection with herpes simplex.

Choice (E): Varicella zoster infection initially starts as a pruritic rash, then later develops into teardrop vesicles, which then ruptures to leave scabs. Usually several stages of lesions (macules, papules, vesicles etc.) are present at the same time.

Option (F): Erysipelas begins as a small erythematous patch that progresses to a red, indurated, tense, and shiny plaque. It usually occurs over the cheek and often there is a history of trauma or pharyngitis. Presence of a ‘raised sharply demarcated’ advancing margins is a classic feature. Local signs of inflammation are universal. Overlying skin streaking and regional lymphadenopathy indicates lymphatic involvement. Over 80% of the cases are due to Streptococci; thus penicillin is the drug of choice.

Educational Objective:Impetigo presents as an erythematous macule, which then rapidly evolves into vesicles and pustules that rupture-leaving honey colored crusted exudates.

48% of people answered this question correctly

Test #18   Q.Id = 655 Question 21 Of 46Item Review

Division : Dermatology

An 83-year-old man is brought for the evaluation of frequent falls for the past 3 months. She also adds that he’s not his usual self these days. He requires help, even to carry on his daily activities. You see an old man with mask like face, talks very softly with poorly modulated voice and has a fine tremor in both of his hands. On examination, he has positive Myerson’ s sign, resting tremor in hands that disappear with voluntary movements, shuffling gait with short steps, stooped posture, tendency to fall and rigidity of both upper limbs. His skin exam shows dry scales with underlying erythema in scalp, central face, presternal region, interscapular areas, umbilicus and body folds. What is the most likely diagnosis of his skin condition?

  A. Tinea versicolorB. Seborrheic dermatitis

  C. Pityriasis rosea  D. Dermatophytosis

E. Lichen simplex chronicusExplanation:

Page 24: Test #18 Q.id = 667

The above said signs and symptoms of rigidity, resting tremors, and bradykinesia are diagnostic features of Parkinsonism. These patients have an expressionless (mask like) face, positive Myerson’ s sign (repetitive tapping over the bridge of the nose produces a sustained blink response), soft voice, micrographia, and a mild decline in intellectual function. The skin condition, which is a common association with Parkinsonism, is seborrheic dermatitis and is characterized by dry scales with underlying erythema in scalp, central face, presternal region, interscapular areas, umbilicus and body folds. These areas may be oily or dry with scales or yellow scurf. The condition is frequently seen in patients with Parkinson’s disease, acutely ill patients who’ve been hospitalized, and HIV positive individuals. Choices A, C, D, E are not associated with Parkinson’s disease.

Choice (A) is tinea versicolor that either appears as pale macules that will not tan or appears as hyper-pigmented macules.

Choice (C) is pityriasis rosea, a common, mild inflammatory disease, which appears as an oval, fawn colored scaly eruption. The centers of lesion have a crinkled appearance.

Choice (D) is a superficial fungal infection of the skin.

Choice (E) is lichen simplex chronicus that is also known as neurodermatitis; it represents a self-perpetuating scratch-itch cycle.

Educational Objective: Seborrheic dermatitis may be seen in association with Parkinsonism and is characterized by dry scales with underlying erythema in scalp, central face, presternal region, interscapular areas, umbilicus and body folds.

38% of people answered this question correctly

Test #18   Q.Id = 679 Question 22 Of 46Item Review

Division : Dermatology

The parents of a 4-year-old girl bring her with complaints of red rash and blisters. According to them, yesterday she had fever and was irritable, and today they noticed the rash with blisters. On examination, you notice superficial flaccid bullae, erythematous rash covering her body diffusely and positive Nikolsky’ s sign. Her face appears edematous and you notice crusting around her mouth area. The skin is warm and tender with exfoliation. Her vital signs are, PR: 90/min; BP: 90/60 mm of Hg; RR: 14/min; Temperature: 39C (102F). What is your diagnosis?

  A. Toxic epidermal necrolysis.B. Staphylococcal scalded skin syndrome.

Page 25: Test #18 Q.id = 667

  C. Scarlet fever.  D. Erysipelas.  E. Impetigo.

Explanation:

Staphylococcal scalded skin syndrome is a disease in children, mostly in those under 5 years old. It is a syndrome of acute exfoliation caused by toxins produced by phage, group 2 staphylococcus aureus. The prodromal infection is usually not apparent. Onset is sudden with a diffuse erythema, skin tenderness and fever, within a day flaccid bullae appears. Facial edema and perioral crusting is often seen. Treatment focuses on stabilizing the patient. They require fluid replacement, topical wound care treated similar to burn wounds and parenteral antibiotics to cover staphylococcus.

Choice (A): Toxic epidermal necrolysis is more benign than staphylococcal scalded skin syndrome. Hot, tender, erythematous morbilliform or discrete macules and loose patches of skin characterize it. The oral mucosa is frequently involved with blistering and erosions. Patients give a history of taking sulfonamides, phenytoin, barbiturates, or NSAIDs.

Option(C): Scarlet fever is caused by group A beta-hemolytic streptococcal infection. Exudative tonsillitis precedes the condition. The rash typically appears as scarlet macules over generalized erythema.

Choice (D): Erysipelas begins as a small erythematous patch that progresses to a red, indurated, tense, and shiny plaque; it usually occurs over the cheek and often there is a history of trauma or pharyngitis. Presence of a ‘raised sharply demarcated’ advancing margins is a classic feature. Local signs of inflammation are universal. Overlying skin streaking and regional lymphadenopathy indicate lymphatic involvement. Over 80% of the cases are due to Streptococci; thus penicillin is the drug of choice.

Choice (E): Impetigo is a contagious infection of the skin caused by staphylococcus or streptococcus or both. The lesions appear as macules, vesicles, and bullae with honey colored crusts.

Educational Objective:Staphylococcal scalded skin syndrome, a disease of children, presents with sudden onset of diffuse erythema, skin tenderness, fever, flaccid bullae, facial edema and perioral crusting.

34% of people answered this question correctly

Test #18   Q.Id = 653 Question 23 Of 46Item Review

Division : Dermatology

A 36-year-old male presents with complaints of darkening and thickening of skin

Page 26: Test #18 Q.id = 667

over the neck and groin area with occasional itching for the past 2 months. On examination, you notice symmetrical, hyperpigmented, velvety plaques in the axilla, groin and posterior neck. His vital signs are, PR: 82/min; BP: 130/80 mm of Hg; RR: 14/min; Temperature: 36.8C(98.4F). Presence of this condition in this patient should alert the physician to check for?

A. Immediate gastrointestinal malignancy workupB. Diabetes mellitus

  C. Addison’s disease  D. Pellagra  E. Hemochromatosis

Explanation:

The skin findings of symmetrical, hyperpigmented, velvety plaques in axilla, groin, and neck are suggestive of acanthosis nigricans. This can be associated with both choices (A) and (B) but insulin resistance is the common cause in the younger age population. Whereas, a gastrointestinal malignancy is a concern in older patients.

Choice (A): Associated with acanthosis nigricans in older individuals.

Choice (C): Hyperpigmentation of the skin over pressure areas is hallmark of Addison’s disease. The other findings are vitiligo, dehydration, and hypotension. Normal BP in this patient, and the characteristic skin findings, is not suggestive of Addison’s disease.

Choice (D): Pellagra is the late stage of Niacin deficiency with a classic triad of dermatitis, diarrhea, and dementia.

Choice (E): Though diabetes mellitus and hyperpigmentation of skin are features of hemochromatosis, the skin findings are different from those of acanthosis. The coloration is brownish or bronze and at times slate gray.

Educational Objective:Acanthosis nigricans is characterized by symmetrical, hyperpigmented, velvety plaques in axilla, groin, and neck; it is associated with diabetes mellitus in younger patients and gastrointestinal malignancy in older individuals.

8% of people answered this question correctly

Test #18   Q.Id = 2040 Question 24 Of 46Item Review

Division : Dermatology

An 18-month-old Caucasian boy is brought to the emergency room with a 3-day history of fever and facial rash. Past medical history is significant for atopic dermatitis diagnosed 1 week ago and treated with topical steroids. Examination

Page 27: Test #18 Q.id = 667

reveals numerous umbilicated vesicles over the erythematous skin on both cheeks. Submandibular adenopathy is also present. What is the most probable diagnosis in this patient?

A. Varicella  B. Impetigo contagiosa  C. Contact dermatitis  D. Atopic dermatitis exacerbation

E. Eczema herpeticumExplanation:

The clinical scenario described is characteristic for eczema herpeticum. Eczema herpeticum is a form of a primary herpes simplex virus infection that is associated with atopic dermatitis. It is usually superimposed on healing atopic dermatitis lesions after the exposure to the herpes simplex virus. Numerous umbilicated vesicles over the area of healing atopic dermatitis are typical. It is frequently accompanied with fever and adenopathy. In infants, the infection may be life threatening, and acyclovir treatment should be initiated as soon as possible.

Varicella (Choice A) is characterized by a vesicular eruption that is not localized, but tends to spread over the head and to the trunk.

Impetigo contagiosa (Choice B) manifests as thick-crusted facial lesions that are frequently honey-colored.

Contact dermatitis (Choice C) rarely presents as vesiculated lesions.

Atopic dermatitis, itself (Choice D), can cause eczematous skin lesions, but the clinical scenario described is not typical for this disorder.

Educational Objective:Eczema herpeticum is a form of a primary herpes simplex virus infection associated with atopic dermatitis. Numerous vesicles over the area of atopic dermatitis are typical. The infection can be life threatening in infants, and prompt treatment should be initiated.

6% of people answered this question correctly

Test #18   Q.Id = 678 Question 25 Of 46Item Review

Division : Dermatology

A 42-year-old woman presents with painful skin lesions. She also complains of inability to eat or drink, as the lesions have involved her mouth and throat. Her other complaints include malaise, headache, sore throat, cough, nausea and vomiting prior to the onset of skin lesions. She was in perfect health in the past,

Page 28: Test #18 Q.id = 667

other than an episode of urinary tract infection 10 days ago. She was prescribed trimethoprim – sulfamethoxazole combination, for this infection. On examination, you notice that the skin is hot, tender with erythematous macules. The picture showing the skin of her back is given here. The oral mucosa shows blistering and erosions. Her vital signs are, PR: 92/min; RR: 14/min; BP: 140/90 mm of Hg; Temperature: 38.3C(101F). What is the most likely diagnosis?

  A. Exfoliative dermatitis.  B. Staphylococcal scalded skin syndrome.

C. Toxic epidermal necrolysis.  D. Erythema multiforme minor.

E. Stevens Johnson’s syndrome.Explanation:

Toxic epidermal necrolysis is a severe mucocutaneous exfoliative disease. The primary manifestation is an erythematous morbilliform eruption that rapidly evolves into exfoliation of skin. This patch of skin slides off on slightest pressure, positive Nikolsky’ s sign and these features are clearly demonstrated in the picture shown above.  Oral mucosa shows painful blisters, which limit oral intake. The exact pathogenesis is not known but it represents an idiosyncratic reaction to drug or chemical agent. The drugs commonly involved are sulfonamides, barbiturates, phenytoin, and NSAIDs. No specific therapy exists. The treatment is supportive to reduce fluid and electrolyte loss and prevent secondary infection.

Choice (A): Exfoliative dermatitis is a scaly dermatitis, causing generalized scaly eruption of the skin; it may be drug induced, idiopathic or secondary to underlying dermatological or systemic disease. Gynecomastia is a common finding in these patients.

Choice (B): Staphylococcal scalded skin syndrome is a syndrome of acute exfoliation of skin caused by toxin produced by phage group 2 staphylococcus aureus. The skin is tender, warm with diffuse erythematous rash, which is sand paper like; bullae, positive Nikolsky’ s sign, facial edema, perioral crusting, and dehydration are other features. It is usually seen in children under 6 years of age.

Choice (D): Erythema multiforme minor lesions can be macular, papular, bullous or purpuric. “Target” lesions can be noted and they favor the extensor surface, palms, soles and mucous membranes.

Choice (E): Stevens Johnson’s syndrome also known as erythema multiforme major, is marked by toxicity and involvement of two or more mucosal surfaces.

Page 29: Test #18 Q.id = 667

Sulfonamides, NSAIDs and phenytoin often cause it. The typical lesions have the appearance of “target” and are pathognomonic.

Educational Objective:Toxic epidermal necrolysis is a severe mucocutaneous exfoliative disease and is characterized by an erythematous morbilliform eruption that rapidly evolves into exfoliation of skin.

17% of people answered this question correctly

Test #18   Q.Id = 670 Question 26 Of 46Item Review

Division : Dermatology

A 15-year-old male is brought with the complaint of sudden onset of difficulty in breathing for the past 45 minutes. He also complains of nausea, colicky abdominal pain and swollen face. He has been suffering from bronchitis for the past 4 days and his condition is now improving. His mother says that he had a similar episode when he had a tooth extraction 2 year ago. On examination, you notice edematous swelling of his face including lips, hands, arms, legs, genitals and larynx. His vitals are as follows: PR: 82/min; BP: 120/80 mm of Hg; RR: 18/min; Temperature: 36.8C(98.4F). Which of the following best explains the pathological process of his condition?

  A. C3 inhibitor deficiency.  B. Antibody mediated hypersensitivity.

C. C1 inhibitor deficiency.  D. Immune complex mediated hypersensitivity.  E. Cell mediated hypersensitivity.

Explanation:

Angioedema can be easily diagnosed from the rapid onset of symptoms which include non-inflammatory edema of face, limbs, genitalia; laryngeal edema; and edema of the bowels resulting in colicky pain. Laryngeal edema can result in life threatening airway compromise. Both hereditary and acquired forms of C1 inhibitor deficiency exist with similar clinical manifestations. The deficiency leads to elevated levels of edema-producing factors C2b and bradykinin. Patients with hereditary angioedema usually present in late childhood. Usually, episodes of angioedema follow infection, dental procedures, or trauma.

Choices (A), (B), (D), (E) are not involved in the pathological process of angioedema.

Choice (B): Antibody mediated hypersensitivity can be seen in immune hemolytic anemia and Rh hemolytic disease of the new born. It involves specific reactions of IgG or IgM antibodies to cell bound antigens leading to complement activation.

Page 30: Test #18 Q.id = 667

Choice (D): Immune complex mediated hypersensitivity can be seen in Arthus reaction and in serum sickness. The antibodies of IgG or IgM forms complexes with allergens, present in tissues, leading to activation of the complement cascade.

Choice (E): Most common example of cell-mediated hypersensitivity is allergic contact dermatitis; allergen causes dermal inflammation on direct contact with skin, after a latent period of 1-2 days.

Educational Objective:Angioedema is characterized by rapid onset of non-inflammatory edema and is due to deficiency of C1 esterase inhibitor which results in elevated levels of edema-producing factors C2b and bradykinin.

39% of people answered this question correctly

Test #18   Q.Id = 671 Question 27 Of 46Item Review

Division : Dermatology

A 50-year-old man was admitted to cardiac intensive care unit 2 weeks ago with complaints of chest pain, profuse sweating and anxiety. His ECG showed ST segment elevation and Q waves in inferior leads. He was taken to catheterization lab and had an angioplasty with stent done for 100% occlusion of right coronary artery. He was discharged with several medications which include aspirin, clopidogrel, metoprolol, enalapril, simvastatin and isosorbide mononitrate. Now he presents with acute onset of respiratory difficulty, periorbital, circumoral, and facial edema. His vital signs are, PR: 82/min; BP: 130/80 mm of Hg; RR 24/min; Temperature: 36.8C(98.4F). Which of the following is most likely to be responsible for his present symptoms?

  A. Metoprolol  B. Isosorbide mononitrate  C. Clopidogrel  D. Aspirin

E. Enalapril  F. Simvastatin

Explanation:

ACE inhibitors are usually started on the first post-infarction day, in non-hypotensive patients. ACE inhibitors are notorious for causing isolated angioedema seen in emergency rooms. Patients present with symptoms of non-inflammatory subcutaneous edema in non-dependent areas and laryngeal edema which can be life threatening. Angioedema is believed to be due to pro-inflammatory action of substance-P. Substance-P release is stimulated by bradykinin. Bradykinin can be broken down in several ways, of which one is

Page 31: Test #18 Q.id = 667

cleavage by angiotensin converting enzyme. When this enzyme is blocked by ACE inhibitors the levels of bradykinin increase, leading to angioedema. Patients typically present within days to weeks, after starting therapy, as in this patient. Risk of recurrence with continuation of the drug is much higher. The patient should first be assessed for airway compromise and vasomotor instability. The presence of which requires subcutaneous epinephrine administration. If airway obstruction fails to respond to epinephrine, emergency tracheostomy is done.

Even though all the above-mentioned drugs can cause allergic reactions, angioedema is not typical for any medication other than enalapril.

Common side effects of beta-blockers (metoprolol) are bradycardia, AV block, and bronchospasm.

Clopidogrel usually causes thrombocytopenia.

Statins are associated with elevated LFTs, and myopathy.

Nitrates can cause headaches, hypotension, and drug intolerance.

Aspirin usually causes gastritis and tinnitus.

Educational Objective:ACE inhibitor is an important cause of angioedema.

24% of people answered this question correctly

Test #18   Q.Id = 666 Question 28 Of 46Item Review

Division : Dermatology

A 28-year-old Caucasian woman comes with the complaints of itching and pain over a mole present on her lower back for the past 2-3 months. She also noticed that the mole is larger with a darker hue than before. On further questioning, she says that she’s sun sensitive and also gives history of sunburns. On examination, you notice a lesion which is shown in the picture below. Which of the following could be the possible diagnosis?

  A. Keratoacanthoma  B. Blue nevi

C. Malignant melanoma  D. Melanocytic nevi  E. Lentigo simplex

Page 32: Test #18 Q.id = 667

Explanation:

Melanoma occurs as solitary lesion anywhere on the skin but frequently over the back and other areas easy to miss on self-inspection. The A, B, C, D, E ‘s of melanoma help in screening and early detection. These letters represent Asymmetry, Border irregularities, Color variegations, Diameter greater than 6mm, and Enlargement. 7 points checklist is designed in England, which consists of 3 major, and 4 minor features, designed to be used by lay people. The 3 major features are change in size, change in color, change in shape. The 4 minor features are inflammation, bleeding or crusting, sensory changes, and lesion of 7mm or more. A changing mole is the most common symptom. Symptoms such as bleeding, itching, ulceration, and pain in a pigmented lesion are less common but needs evaluation. The risk factors are sun sensitive skin, immunosuppression, xeroderma pigmentosa, family history of melanoma, dysplastic mole syndrome, and atypical nevi. The lesion shown in the above picture is raised, black in color with irregular border and most likley is malignant melanoma.

Keratoacanthoma is a low-grade malignancy that pathologically resembles squamous cell carcinoma. It appears as a solitary, firm, round, skin colored or reddish plaque that develops into a nodule with central keratin plug.

Blue nevi are smooth surfaced dome shaped papules that develop from a macule to papule, tend to be less than 1cm, and are due to arrest in the migration of neural crest melanocytes.

Melanocytic nevi are common lesions found in the integument of most individuals and have a malignant potential.

Lentigo simplex is not induced by sun exposure and not associated with systemic disease. Clinically the lesions are round or oval macules with even pigmentation.

Educational Objective:Whenever a patient presents with a changing mole, always suspect malignant melanoma.

63% of people answered this question correctly

Test #18   Q.Id = 649 Question 29 Of 46Item Review

Division : Dermatology

A 48-year-old white male comes to see you in the emergency department with complaints of severe pain and swelling in his left leg. He sustained an injury to his left leg while playing tennis, 5 days ago. He complains of pain in the area of injury and flu like symptoms. He says that his pain worsened over the past 2-3 days and is unbearable now. On examination, you notice edematous limb with a

Page 33: Test #18 Q.id = 667

purplish discoloration of the injured area with bullae and a serosanguineous discharge. The leg is extremely tender to touch. A scalpel incision of the skin reveals yellowish green necrotic tissues. His vitals are, PR: 104/min; BP: 110/80 mm of Hg; RR: 18/min; Temperature: 39C(102F). What is your diagnosis?

  A. Thrombophlebitis  B. Cellulitis

C. Necrotizing fasciitis  D. Erythema induratum  E. Toxic shock syndrome

Explanation:

Necrotizing fasciitis is a rapidly spreading infection involving the fascia of deep muscles. It usually occurs after trauma or around foreign bodies in surgical wounds or idiopathic as in scrotal or penile necrotizing fasciitis. Group A hemolytic Streptococci and Staphylococcus aureus alone or in synergism are frequently the initiating infectious bacteria but, other aerobic and anaerobic pathogens may be present. Typically, there is a sudden onset of pain and swelling at the site of trauma or recent surgery. A history of diabetes mellitus should be suspected. As the condition progresses there is purplish discoloration with gangrenous changes. A gloved finger can easily be passed between the 2 layers and reveals a yellowish green necrotic fascia, which helps in the diagnosis. Signs of systemic toxicity such as fever and hypotension may be present. CT scanning pinpoints the involved site by showing necrosis with asymmetrical fascial thickening and presence of gas in the tissues. Aggressive treatment is necessary to reduce morbidity and mortality. Thorough surgical debridement of all the necrotic tissues is the most important therapy. High flow oxygen, fluid resuscitation, and broad-spectrum antibiotics should be part of management.

Choice (A): Thrombophlebitis is characterized by palpable, indurated, cord like, tender, subcutaneous venous segments. Fever is usually low grade.

Choice (B): Cellulitis is an acute spreading infection of the dermis and subcutaneous tissue. The hallmarks of cellulitis are warmth, erythema, edema, and tenderness. Necrotic changes in deeper tissues are not seen.

Choice (D): Erythema induratum appears as a nodular eruption in patients with tuberculosis. On examination, there are crops of small, tender, erythematous nodules involving shins and calves.

Choice (E): Toxic shock syndrome is characterized by invasion of skin and soft tissues, acute respiratory distress, and renal failure. Infants, elderly, and those with underlying medical conditions are at risk for a very invasive disease.

Educational Objective:

Page 34: Test #18 Q.id = 667

In necrotizing fasciitis, there is purplish discoloration of skin with gangrenous changes as well as systemic signs of toxicity. Differentaite it from cellulitis.

55% of people answered this question correctly

Test #18   Q.Id = 662 Question 30 Of 46Item Review

Division : Dermatology

A 36-year-old male, a known AIDS patient, comes in for his painful red eye. He says that he has been having eye pain, discharge, and redness in his left eye for the past 10 days. On physical examination, you notice redness in his left eye, multiple new skin lesions on his face, left eyelid, inner thighs, penis and pubis, which were not present on previous visit. The lesions are painless, pale, and shiny, dome shaped, small around 2-5mm, with central umbilication. His CD4 count was 100/uL, 30 days ago. What is the etiology for his skin lesions?

  A. Human herpes virus 8  B. Staphylococcus

C. Poxvirus  D. Herpes simplex 1  E. Human papilloma virus

Explanation:

The above described multiple dome shaped lesions, with central umbilication, is highly suggestive of molluscum contagiosum caused by Poxvirus. It’s usually disseminated in immunodeficient conditions, such as AIDS, especially when the CD4 count is less than 100/uL. A chronic conjunctivitis, as in this patient, may occur if it is located in the lid margins. Molluscum resolves spontaneously in one year. Treatment options are simple excision, cryotherapy or desiccation.

Choice (A) is also known as KSHV or Kaposi’s sarcoma associated herpes virus (HHV8). These lesions are reddish purple, dark plaques or nodules on cutaneous or mucosal surfaces.

Choice (B) does not cause the above-explained lesions.

Choice (D) may cause corneal ulceration and vesicles on skin but not dome shaped central umbilicated skin leasions.

Choice (E) causes skin warts, which are verrucous papules anywhere on the skin or mucous membranes.

Educational Objective: Molluscum contagiosum, characterized by multiple dome shaped lesion, with central umbilication, is caused by poxvirus and is an opportunistic skin infection

Page 35: Test #18 Q.id = 667

in HIV-infected patients.

40% of people answered this question correctly

Test #18   Q.Id = 647 Question 31 Of 46Item Review

Division : Dermatology

A 30-year-old, Caucasian male, presents with complaints of pale patches over his trunk area, in a mottled distribution. He had just returned from a 2-week summer vacation to the Bahamas, where he first noticed these lesions. His skin is well tanned in general, with multiple, velvety pink, pale macules, measuring around 4-5mm in diameter. These lesions scale on scraping. The lesions are located over his central upper trunk area. What is the diagnosis of his condition?

  A. Vitiligo  B. Seborrheic dermatitis

C. Tinea versicolor  D. Pityriasis rosea  E. Tinea corporis

Explanation:

This eruption often comes to the patient’s attention by the fact that the involved areas never tan and the hypopigmentation is often considered as vitiligo. Pale, velvety pink or whitish, macules that do not tan and do not appear scaly but scale on scraping are the features suggestive of tinea versicolor. It is caused by Malassezia furfur, a superficial fungal infection of the skin. Microscopic examination of the skin scraping, after KOH preparation reveals large, blunt hyphae and thick walled budding spores called as “spaghetti and meatballs” appearance. Sometimes it presents as hyper-pigmented macules velvety tan or brownish; this form is not so uncommon. Topical treatment with selenium sulfide lotion and ketoconazole shampoo can be used. The change in pigmentation requires months to get back to normal.

Choice (A) Vitiligo usually presents with peri-orificial lesions or lesions on the tips of fingers. It is characterized by total depigmentation and not just lessened pigmentation, as in tinea versicolor.

Choice (B) Seborrheic dermatitis is acute or chronic papulosquamous dermatitis, characterized by dry scales and underlying erythema. Scalp, central face, presternal, interscapular area, umbilicus, and body folds are usually involved. Pigmentation changes are not seen.

Choice (D) Pityriasis rosea lesions appear as oval, fawn colored plaques that measures upto 2 cms in diameter and occur in a Christmas tree pattern. The initial lesion is called the herald patch, which is followed by a generalized

Page 36: Test #18 Q.id = 667

eruption in 1-2wks.

Choice (E) or tinea corporis, body ringworm presents as ring-shaped lesions with an advancing scaly border and central clearing or as scaly patches over the trunk.

Educational Objective:Pale, velvety pink or whitish, hypopigmented macules that do not tan and do not appear scaly are suggestive of tinea versicolor.

42% of people answered this question correctly

Test #18   Q.Id = 680 Question 32 Of 46Item Review

Division : Dermatology

A 25-year-old woman comes to see you with the complaint of pale patches of skin around her mouth, which she noticed a few months ago and they have become more prominent now. There is no itching, burning, or numbness over the patches. On examination, you notice pale white patches symmetrically placed around her mouth. The borders of these macules are well circumscribed and hyper-pigmented. Similar lesions were also found over the areola of breasts. She denies any history of trauma or infection. Her vital signs are stable and she is afebrile. Which of the following best explains the pathology of her condition?

  A. Post inflammatory.B. Destruction of melanocytes.

  C. Inherited absence of melanocytes.  D. Infection with mycobacterium leprae.  E. Superficial fungal infection.

Explanation:

Vitiligo is a specific form of leukoderma, usually peaks in persons aged 20-30 years. The depigmentation has a predilection for acral areas and around body orifices as in this patient. It appears as pale whitish macules the borders of which are hyper-pigmented. The condition is slowly progressive; few experience spontaneous remission. The pathogenesis is thought to be an autoimmune process, which causes destruction of melanocytes.

Choice (A): Post inflammatory hypo-pigmentation is usually associated with a history of trauma or inflammation. There is a reduction in pigmentation rather than complete absence as in this patient.

Option(C): Inherited absence of melanocytes is also known as piebaldism. It is usually noticed at birth and is confined to head and trunk.

Page 37: Test #18 Q.id = 667

Choice (D): Infection with mycobacterium leprae causes leprosy. Areas of hypo pigmentation with anesthesia characterize it.

Choice (E): Superficial fungal infection can result in pale macules due to lessening of pigmentation rather than total absence as in vitiligo. These macules can be velvety pink or whitish; sometimes there are hyper-pigmented patches.

Educational Objective:Vitiligo presents with depigmentation in the form of macules and it involves acral and peri-orificial areas and is caused by autoimmune destruction of melanocytes.

42% of people answered this question correctly

Test #18   Q.Id = 1679 Question 33 Of 46Item Review

Division : Dermatology

An 18-year-old white female who is alert but listless was brought to the ER due to severe vomiting, fever and rash. She was fine until today when she developed a fever, flu like symptoms and dizziness. She has a history of asthma and allergic rhinitis, and denies taking drugs or alcohol, or being exposed to other sick individuals. She had her periods yesterday but didn’t place a tampon until today, 6 hours prior to becoming ill. Her last menstrual period was 6 weeks ago. On examination, there is an erythematous, flat and raised rash on her trunk and extremities. Her vitals are T: 38.8 C, PR: 120/min, RR: 23/mn and BP: 100/66 mm of Hg supine and 66/30 mm of Hg standing. Based on this history and physical examination what is the most likely diagnosis?

A. Toxic shock syndrome  B. Scarlet fever  C. Meningococcemia  D. Stevens-Johnson syndrome  E. Toxic epidermal necrolysis

Explanation:

Toxic shock syndrome (TSS) is a potentially fatal condition caused by toxins produced by specific strains of staphylococci, including epidermal exfoliating toxin. TSS was initially found to occur in menstruating women using tampons, but it has been later correlated with other intravaginal articles such as diaphragms, sponges, cervical caps, or rarely, post-operative wound infections such as episiotomy. Symptoms include a sudden onset of a flu-like syndrome, high fever, hypotension, erythematous rash, and signs of systemic failure. Blood cultures are usually negative, and increased creatinine/BUN, as well as low platelet count may be noted. About 1 to 2 weeks after onset, peeling of skin surfaces, including palms and soles usually ensue. TSS should be detected early and managed vigorously. Patients should be hydrated and debridement of

Page 38: Test #18 Q.id = 667

the wound should be performed; all sources of infection should be removed and anti-staphylococcal antibiotics should be started immediately, although they usually do not shorten the course of the illness. For prevention, women should be advised to change tampons frequently and informed about the associated risk of TSS with barrier intravaginal methods; they should also be instructed about the clinical signs for early recognition and prompt management.

Option B: Scarlet fever is a febrile illness caused by some strains of streptococcus that produce an erythrogenic toxin. It is usually preceded by pharyngitis, and manifests with high fever, erythematous "sandpaper-like" rash, usually located on the chest, and pharyngitis.

Option C: Meningococcemia occurs following untreated meningococcal meningitis. It is characterized by a disseminated necrotic purpura, altered mental status and shock.

Option D: Stevens-Johnson syndrome is a severe hypersensitivity reaction of the skin and mucous membranes, generally secondary to drugs. Patients present prodromes of fever, malaise, myalgias, arthralgias, headaches, emesis and diarrhea. Between 1 to 14 days later, there is a sudden onset of high fever, extensive lesions such as macules, papules and bullas on the skin and mucous membranes; GI, respiratory, or genitourinary tracts may also be involved. Mortality is about 10% if the condition is untreated.

Option E: Toxic epidermal necrolysis is the most severe form of cutaneous hypersensitivity. It manifests with diffuse erythema, burning, bulla formation and peeling of the epidermis causing widespread exfoliation. Mucous membranes may also be involved and multiple organ failure, involving the kidneys and liver, may ensue.

Educational objective:Toxic shock syndrome (TSS) is a potentially fatal condition caused by toxins produced by specific strains of staphylococci, including epidermal exfoliating toxin and is related to females using tampons and other intravaginal articles.

57% of people answered this question correctly

Test #18   Q.Id = 2027 Question 34 Of 46Item Review

Division : Dermatology

A 17-year-old white male presents to your office with a several month history of acne. He says that, ‘the spots on the face got worse,’ recently. He has not visited a doctor before. Inspection reveals multiple papules with several pustules and nodules. Atrophic scars are also seen. What is the next best step in the management of this patient?

Page 39: Test #18 Q.id = 667

  A. Topical retinoids  B. Topical antibiotic  C. Oral antibiotic  D. Benzyl peroxide

E. Oral isotretinoinExplanation:

Isotretinoin is a very effective agent in treating a nodulocystic-type of acne. It is an oral retinoid that greatly diminishes sebum excretion and is active against Propionibacterium acne. At the same time, it has several serious side effects (the most famous is teratogenicity), and it is not equally effective against all types of acne. The main indications for isotretinoin therapy are recalcitrant or severe cases of nodulocystic acne and resistant cases of moderate acne. It also should be promptly instituted in patients who have scars.

(Choice A) Topical retinoids are the initial drugs of choice for noninflammatory comedones-type acne.

(Choice B) Mild inflammatory acne is treated with topical benzyl peroxide or a topical antibiotic (erythromycin, clindamycin, tetracycline) or a combination of two.

(Choice C) Oral antibiotics (most commonly tetracyclines) are used in moderate-to-severe inflammatory acne.

Educational Objective:In patients with moderate-to-severe acne with predominantly a nodulocystic form and who have developed scars, should be treated with oral isotretinoin.

*Extremely high yield question for USMLE!!!

9% of people answered this question correctly

Test #18   Q.Id = 663 Question 35 Of 46Item Review

Division : Dermatology

A 28-year-old Caucasian female, a lifeguard at the local beach, comes in with complaints of a small swelling on her left lower eyelid for the past 6 months. She has recently noticed loss of eyelashes of the lower eyelid. On examination, there is a small nodular lesion on the lower eyelid margin. It is firm, painless, pearly and indurated. There is loss of lashes of the left lower eyelid. Based on the above presentation, which of the following is the most probable diagnosis?

  A. Squamous cell carcinoma  B. Keratoacanthoma

C. Basal cell carcinoma  D. Squamous papilloma

Page 40: Test #18 Q.id = 667

  E. Seborrheic keratosisExplanation:

Basal cell carcinoma is the most common malignant tumor of the eyelid. It usually occurs in fair skinned individuals with history of prolonged sun exposure. These are usually slow growing, pearly, and indurated. Lower eyelid margin is the most common location followed by medial canthus, upper eyelid, and lateral canthus. These tumors are locally invasive and rarely metastasize. There are two modes of therapy for basal cell carcinoma in the eyelid, chemosurgery and excision with frozen section control.

Choice (A). Squamous cell carcinoma is much less common and faster growing. It presents as nodules or plaques with everted edges.

Choice (B). Keratoacanthoma is a rapidly growing hyper-keratotic nodule with a central keratotic plug.

Choice (D). Squamous papilloma is a most common benign tumor of the eyelid, caused by HPV. It presents as a frond-like or lobular projection.

Choice (E). Seborrheic keratosis appears as greasy, brownish crust-like lesions with a stuck on appearance.

Educational Objective: Basal cell carcinoma is the most common malignant tumor of the eyelid and presents as slow growing, pearly, and indurated lesion.

47% of people answered this question correctly

Test #18   Q.Id = 29 Question 36 Of 46Item Review

Division : Dermatology

A 36-year-old Caucasian male comes to you 3 weeks after being medically treated for bipolar disorder by his psychiatrist. He complains of having a “rash” for the last few weeks. He thinks that sunbaths, which he has been taking for the last few weeks, might be the cause of precipitating the skin disease. His other medications include ibuprofen for joint pain, and over-the-counter vitamin supplements. On examination, PR: 86/min; BP: 130/80 mm Hg; RR: 18/min. He is afebrile. His skin rash is shown in the picture below. What factor is most likely responsible for his symptoms?

  A. Sunbaths   B. Ibuprofen   C. Over the counter vitamins

Page 41: Test #18 Q.id = 667

  D. Gabapentin   E. Lamotrigine   F. Carbamazepine

G. Lithium H. Valproic acid

  I. Fluoxetine   J. Phenytoin

Explanation:

The patient is most likely suffering from psoriasis. The image given above clearly shows red, sharply defined plaques covered with whitish scales over knees and is typical of psoriasis. The history suggests that patient is also suffering from psoriatic arthritis and has been taking Ibuprofen, which may have been prescribed to him by a physician earlier. Lithium may precipitate or exacerbate psoriasis. Ibuprofen, on the other hand, increases lithium blood levels which will further increase the chance of psoriasis exacerbation.

Choice (A). Sunbath is highly unlikely as ultra-violet light exposure is used as part of treatment for psoriasis.

Choice (B). Phenytoin may also be used in treatment of bipolar disorder in addition to its indication in epilepsy. Its side effects include: gingival hyperplasia; hirsutism; lymphadenopathy; nystagmus; ataxia; and megaloblastic anemia.

Choice (C). Ibuprofen doesn’t itself exacerbate or precipitate psoriasis, however it will increase lithium levels.

Choice (D). Over-the-counter vitamin supplements do not cause exacerbation of psoriasis.

Choice (E). Gabapentin is an antiepileptic and sometimes used in bipolar disorder. Its main side effects are sedation, behavioral changes especially in children, nystagmus, and weight loss.

Choice (F). Lamotrigine does cause disease with skin manifestations - Steven Johnson Syndrome, which is a potentially life-threatening condition. However, the patient’s clinical picture is not compatible with this diagnosis.

Choice (G). Carbamazepine may be used in bipolar disorder but it doesn’t exacerbate psoriasis. Side effects include nystagmus, diplopia, hepatotoxicity, blood dyscrasia, and hyponatremia.

Choice (I). Valproic acid is an important drug used in the treatment of bipolar disorder and its main side effects include nausea, vomiting, diarrhea, weight gain, alopecia, hepatotoxicity, and teratogenicity.

Page 42: Test #18 Q.id = 667

Choice (J). Fluoxetine is a SSRI and sometimes used in bipolar disorders. It doesn’t exacerbate psoriasis.

Educational Objective:Lithium precipitates and exacerbates psoriasis which is characterized by sharply defined erythematous areas with whitish, silvery and scaly plaques.

28% of people answered this question correctly

Test #18   Q.Id = 684 Question 37 Of 46Item Review

Division : Dermatology

A 20-year-old woman presents with complaints of a rash for the past 2 days. She was in good health until 5-6 days ago when she developed fever, malaise and headache. The rash first appeared on her face, and then rapidly spread to her trunk and extremities. On examination, you notice pink maculopapular rash involving her face, trunk and extremities. Tender lymph nodes are palpable in posterior auricular and posterior cervical areas. Her soft palate reveals patchy erythema. Her vital signs are, PR: 86/min; BP: 110/70 mm of Hg; RR: 14/min; Temperature: 37.2C(99F). What is the most likely diagnosis?

  A. Secondary syphilis.B. Rubella.

  C. Kawasaki disease.  D. Rocky mountain spotted fever.  E. Erythema multiforme.

Explanation:

RNA toga virus, transmitted via respiratory droplets, causes rubella. The disease progresses in the following order:

Exposure to virus; development of fever, malaise, tender sub occipital adenitis after 14-21 days of exposure; 1-5 days later rash appears on the face then it involves trunk and extremities, lasts for less than 3 days. The rash is maculopapular. Polyarthralgia is common in adults. Posterior cervical and posterior auricular lymphadenopathy is common. The risk to the fetus is highest if the mother contracts rubella in the first trimester. Fetal infection leads to congenital rubella syndrome. Few patients develop encephalopathy. Live attenuated rubella vaccine offers best protection against infection. Patients can be treated with acetaminophen for symptomatic relief. The patients can be infectious from 1 week prior to the onset of rash, to 15 days after.

Choice (A): Secondary syphilis usually presents as localized or diffuse mucocutaneous rash and generalized tender lymphadenopathy. The rash may be macular, pustular or mixed; it characteristically presents over the palm and soles, which are not involved in rubella. The papules coalesce to form

Page 43: Test #18 Q.id = 667

condylomata lata, which is highly suggestive of secondary syphilis.

Choice (C): Kawasaki disease is a multisystem disease, also known as mucocutaneous lymph node syndrome. High fever, bilateral conjunctivitis, injected pharynx, cracked lips, strawberry tongue, edema, desquamation of extremities, polymorphic rash, and cervical lymphadenopathy characterize the disease. A specific prodrome is not seen. It usually occurs in children under 10 years of age and is very rarely seen in adults.

Choice (D): Rocky Mountain spotted fever is characterized by a red macular rash, which starts over the wrists and ankles and then spreads centrally. Influenza-like prodrome occurs.

Choice (E): Erythema multiforme is characterized by the sudden onset of erythematous rash with target lesions. It usually follows infection with herpes simplex.

Educational Objective:Rubella is characterized by maculopapular rash, posterior cervical and posterior auricular lymphadenopathy and polyarthralgia.

60% of people answered this question correctly

Test #18   Q.Id = 2087 Question 38 Of 46Item Review

Division : Dermatology  

A 35-year-old Caucasian male underwent bone marrow transplantation from a HLA-matched sibling because of aplastic anemia. Two weeks after the procedure, a maculopapular pruritic rash developed predominantly on the face, hands, and feet. He also complains of diarrhea. The stool is positive for occult blood. Liver function tests are abnormal. Which of the following is the most likely pathophysiologic mechanism of this patient’s condition?

A. Activation of the donor T lymphocytes  B. Activation of the donor B-lymphocytes  C. Activation of the host T lymphocytes  D. Virus-induced lymphocyte proliferation  E. Depression of the donor myelopoiesis

Explanation:

The clinical scenario described is typical for the acute graft-versus-host disease (GVHD). GVHD is common after bone marrow transplantation. Up to 50% of the patients with bone marrow transplantation from matched siblings develop the disease. The target organs for GVHD are the skin (a maculopapular rash involving palms, soles, and face that may generalize is typical), intestine (blood-

Page 44: Test #18 Q.id = 667

positive diarrhea), and liver (abnormal liver function tests and jaundice). The basic pathophysiologic mechanism involved in GVHD is the recognition of the host major and minor HLA-antigens by the donor T-cells and cell-mediated immune response. The donor B-lymphocytes (Choice B) has less importance in the development of GVHD. Activation of the host T lymphocytes (Choice C) mediates the rejection of the graft; and, as a result, the depression of the myelopoiesis (Choice E). Failure to generate blood cells manifests as severe neutropenia for several consecutive days and increased risk of infection. Graft rejection may occur in up to 20% of high-risk patients.

Virus-induced lymphocyte proliferation due to Epstein-Barr virus (Choice D) is the mechanism responsible for the development of immunoblastic lymphoma in patients who underwent bone marrow transplantation.

Educational Objective:GVHD is caused by the recognition of the host major and minor HLA-antigens by the donor T-cells and cell-mediated immune response. The organs typically affected include skin, intestine, and liver.

37% of people answered this question correctly

Test #18   Q.Id = 660 Question 39 Of 46Item Review

Division : Dermatology

A 43-year-old woman presents with complaints of itching all over her body for the past 4 months. She is tired of using over-the-counter products with no relief. So, she decided to see a physician. On further questioning she complaints of loose stools, hard to flush for the past 2 months. On examination she is jaundiced with hepatomegaly and splenomegaly. The lab findings are serum alkaline phosphatase - 200u/dL, serum cholesterol - 250mg/dL, serum bilirubin -3.3mg/dL and serum IGM 450mg/dL. Anti-mitochondrial antibodies were present in the serum. Which of the following benign nodular lesions of the eye is frequently associated with her condition?

  A. Chalazion  B. Hordeolum  C. Molluscum contagiosum

D. Xanthelasma  E. Stye

Explanation:

The above discussed features of pruritus, jaundice, steatorrhea, hepatosplenomegaly with increased levels of alkaline phosphatase, serum bilirubin, cholesterol, and presence of anti mitochondrial antibodies are all suggestive of primary biliary cirrhosis. The disease is progressive and complicated by xanthomas, xanthelasma, osteoporosis, osteomalacia and

Page 45: Test #18 Q.id = 667

pulmonary hypertension. Xanthelasma are soft cholesterol filled yellow plaques that appear on the medial aspects of eyelids bilaterally. These lesions are classic features of primary biliary cirrhosis. These are benign lesions and the therapy is always for cosmetic purposes. Lipid lowering drugs can cause minimal regression, although not consistently.

Choice (A): Chalazion is inflammatory lesion of the meibomian tear glands and is usually seen in patients with rosacea and in those with blepharitis.

Choice (B): Hordeolum is a purulent inflammation of the eyelid and is usually caused by staphylococcus.

Choice (C): Molluscum Contagiosum is caused by poxvirus and is usually seen in immunodeficient patients.

Choice (E): Stye is another name for external hordeolum.

Educational Objective:Xanthelasma, cholesterol filled yellow plaques that appear on the medial aspects of eyelids bilaterally, may occur in the setting of primary biliary cirrhosis.

62% of people answered this question correctly

Test #18   Q.Id = 668 Question 40 Of 46Item Review

Division : Dermatology

A 62-year-old woman comes with a pigmented lesion on the left forearm that does not bother her except for occasional itching. She decided to see you at her daughter’s persuasion. On examination you notice a slightly elevated, brown-colored, lesion with irregular borders and it measures around 7mm. What will be the next step in management?

  A. Shave biopsyB. Excisional biopsy

  C. Dermoscopy  D. Immunohistochemical staining for lineage  E. Incisional biopsy

Explanation:

Her clinical features are all suspicious of malignant melanoma. The first step towards diagnosis will be skin biopsy for histopathological analysis. Excisional biopsy with narrow margins is preferred as it helps to study the tumor depth (Breslow’ s classification), ulceration, level of invasion (Clark level), presence of mitosis, regression, lymphatic and vascular involvement, and host response. Wider margins of more than 1cm may disrupt afferent cutaneous lymph flow and the ability to identify sentinel nodes. So, wider margins are not usually

Page 46: Test #18 Q.id = 667

recommended.

Incisional biopsy is usually indicated when the suspicion for melanoma is low or if the melanoma is larger where excision is not practical.

Shave biopsy should not be used because partial removal of the primary melanoma does not provide accurate depth measurement, which is the most important prognostic factor.

Dermoscopy is a special magnifying device used to evaluate pigmented lesions. It may be used in patients with multiple lesions to help select suspicious lesions for biopsy.

Immunohistochemical staining for S-100 may be helpful in cases, which require differentiation from melanoma simulators.

Educational Objective:Excisional biopsy with narrow margins is the preferred study for the diagnosis of malignant melanoma.

57% of people answered this question correctly

Test #18   Q.Id = 672 Question 41 Of 46Item Review

Division : Dermatology

An 18-year-old girl comes with the complaint of sudden onset of redness and swelling of skin over exposed areas. She spent 2-3 hours at the beach when she started experiencing a burning sensation, followed by redness and swelling of those areas. Her face has moderate inflammatory acne lesions, for which she says, she is on medication for the past 1 month, or so. On examination, you notice erythema, edema and vesicles on her face, neck, hands and chest. Her vitals are as follows: PR: 82/min; BP: 120/80 mm of Hg, RR: 14/min; Temperature: 37.2C(99F). Which of the following best explains her condition?

  A. Allergic dermatitis  B. Erythromycin induced phototoxicity  C. Epidermolysis bullosa

D. Tetracycline induced photosensitivity  E. Benzyl peroxide induced phototoxicity

Explanation:

This patient shows classic features of photosensitivity that include a history of sun exposure and erythema, edema, and vesicles over sun-exposed areas. The oral antibiotics of choice for treatment of inflammatory acne are tetracycline and erythromycin. Tetracycline can cause photosensitivity, as a result of combined effects of chemical and light. This occurs because of the damaging effects of

Page 47: Test #18 Q.id = 667

light-activated compounds on cell membranes and DNA. Typically, reactions appear as exaggerated sunburn. The mainstay of treatment is avoidance of sun, use of sun- protection, and symptomatic treatment. Topical steroids and cool compresses often alleviate the condition.

Choice (A): Allergic dermatitis occurs on contact with the suspected agent. It presents as erythema, edema with pruritus, later followed by vesicles in the area of contact.

Choice (B): Erythromycin can cause GI upset and cholestatic jaundice but usually does not cause photosensitivity.

Choice (C): Epidermolysis bullosa is an inherited bullous disorder, which presents at birth or shortly after it.

Choice (E): The common side effects of benzyl peroxide include irritation, contact dermatitis, dryness, erythema, peeling, and stinging. Photosensitive reactions are usually rare.

Educational Objective:Tetracyclines are an important cause of photosensitivity that appears after sun exposure with erythema, edema, and vesicles over sun-exposed areas.

51% of people answered this question correctly

Test #18   Q.Id = 1747 Question 42 Of 46Item Review

Division : Dermatology

A 30-year-old female comes to you with a circumferential pruritic rash over her right wrist for the last two days. She says that she never had such a complaint before. On further questioning, she adds that she  bought a new bracelet for her right wrist two weeks back and had been wearing it since. Which of the following metals in that jewelry is most likely to cause such symptoms?

  A. Copper  B. Silver  C. Gold  D. Platinum

E. NickelExplanation:

This patient is having  an allergic contact dermatitis secondary to the bracelet that she has been wearing. Allergic contact dermatitis is a type IV hypersensitivity reaction which occurs upon contact with a particular substance. The initial sensitization requires ten to fourteen days and upon re-exposure, the antigen is presented to sensitized lymphocytes which release cytokines and

Page 48: Test #18 Q.id = 667

lead to inflammation within twelve to forty-eight hours.

Plant urushiol found in poison ivy, poison oak, poison sumac, ginkgo fruit and the skin of mangos is the most common sensitizer in North America. However, other common sensitizers include nickel found in jewelry, formaldehyde found in clothing and nail polish, certain fragrances, preservatives, rubber and chemicals in shoes. Allergic contact dermatitis generally presents with an intensely pruritic rash at the area of contact. Treatment includes avoidance of further exposure to the trigger. Other options include calamine lotion, topical antihistamines, topical corticosteroids, and even oral steroids in severe cases.

Educational Objective: Poison ivy and the nickel jewelry can cause allergic contact dermatitis, a type IV hypersensitivity reaction.

53% of people answered this question correctly

Test #18   Q.Id = 675 Question 43 Of 46Item Review

Division : Dermatology

A 25-year-old male presents with complaints of skin lesions over his elbows, knees and neck with intense itching and burning sensation for the past 10 days. He was advised to follow a gluten free diet on his previous visit but he is not compliant with gluten free diet. On examination, you notice flesh colored to erythematous vesicles distributed over the extensor aspects of elbows, knees, posterior neck and shoulders. Some of these lesions are shown in the picture here. His vital signs are stable. Which of the following is the drug of choice for his skin condition?

  A. High potency steroids.B. Low dose acyclovir.

  C. 1% lindane solution.  D. Azathioprine.

E. Dapsone.Explanation:

The signs and symptoms are suggestive of dermatitis herpetiformis; celiac sprue is often an associated condition in these patients. Dapsone is the drug of choice in this condition. Improvement to dapsone is also considered as a diagnostic feature of this condition. The skin lesions start to heal within hours after beginning the drug.

Options (A), (B), (C), and (D) are not useful in the treatment of this condition.

Page 49: Test #18 Q.id = 667

Choice (B): Acyclovir is an antiviral drug used in the treatment of HSV and HZV infections.

Choice (C): 1% lindane solution is used as a topical agent in treatment of scabies.

Choice (D): Azathioprine is an immunosuppressive agent, used in blistering autoimmune diseases like pemphigus vulgaris.

Educational Objective:Dapsone is an effective treatment of dermatitis herpetiformis.

13% of people answered this question correctly

Test #18   Q.Id = 654 Question 44 Of 46Item Review

Division : Dermatology

A 20-year-old feamle comes with the complaints that her skin is always rough, dry and scaly. She says that her skin was normal at birth and then it gradually became dry. The dryness increases during winter months, even though she applies body-moisturizing lotion regularly. On examination you notice the lesions shown in the image below.  What is your diagnosis?

  A. Atopic dermatitis  B. Irritant contact dermatitis

C. Ichthyosis vulgaris  D. Impetigo  E. Psoriasis

Explanation:

The history of normal skin at birth, with gradual progression to dry scaly skin, is typical of ichthyosis. This condition can be hereditary or acquired. The skin is usually dry and rough with horny plates over the extensor surfaces of limbs. The above picture shows all these typical features of ichthyosis. The condition worsens in the winter because of incresaed dryness and sometimes refered as “lizard skin”. There is relative sparing of face and diaper area in children.

Choice (A): Usually occurs in infancy and pruritus is the only symptom and there is involvement of cheeks, forehead, and limbs.

Choice (B): Usually, there is a history of exposure to irritant substance.

Page 50: Test #18 Q.id = 667

Choice (D): Skin has blisters that enlarge and rupture.

Choice (E): Usually presents as silvery scales over the extensor surfaces of elbows, knees, scalp, and trunk.

Educational Objective: Dry and rough skin with horny plates over the extensor surfaces of limbs is hallmark of ichthyosis.

42% of people answered this question correctly

Test #18   Q.Id = 28 Question 45 Of 46Item Review

Division : Dermatology

A 35-year-old white male presents with high-grade fever with chills, rigors, malaise, and pain in his right calf for the last 24 hours. His vitals are, Temperature: 39.5C(103.1F); PR: 105/min; BP: 130/80; RR: 15/min. Examination shows generalized swelling of the calf with linear streaks of erythema. The lesion is warm, tender, and not very well demarcated. No pain is felt in the calf when the ankle is dorsi flexed. Scaling is found in the toe webs on right side and KOH preparation of these lesions show hyphae. Blood cultures were obtained. CBC showed a WBC count of 14,000 with 6% bands. What is the next most appropriate step in the management?

A. Oral itraconazole  B. Oral terbinafine

C. IV nafcillin  D. Oral dicloxacillin  E. IV aqueous crystalline penicillin G

Explanation:

Cellulitis is the inflammation of skin which may extend into the deeper tissues. Beta-hemolytic streptococci mostly cause it but staphylococcus aureus is also a common cause. Clinical features are systemic as well as local. Systemic signs of toxicity are high-grade fever with rigors and chills, malaise, fatigue, and confusion. Local findings are generalized swelling which is erythematous, warm, tender, and less well demarcated than erysipelas. Tinea pedis may be a portal of entry for lower leg cellulitis.

Antifungal agents like itraconazole and terbinafine may reduce the recurrence of lower leg cellulitis when tinea pedis infection coexists but they would not ameliorate the toxicity in acute cases.

When systemic signs of toxicity are present, IV nafcillin is the preferred treatment for cellulitis.

Page 51: Test #18 Q.id = 667

Oral dicloxacillin can be used for the acute treatment of cellulitis when it is mild with no systemic signs.

Many cases of cellulitis are due to staphylococcus aureus; therefore IV aqueous crystalline penicillin G (almost 99% resistance) is not an effective treatment for cellulitis.

Educational Objective:IV nafcillin is the treatment of choice for cellulitis with systemic signs.

33% of people answered this question correctly